Which one of the following inferences is most supported by the information in the passage?

amf on June 1, 2020

hi can you help?

Hi, can you please explain why C would be incorrect here?

Reply
Create a free account to read and take part in forum discussions.

Already have an account? log in

SamA on June 4, 2020

Hello @amf,

I see that you have done the math with answer choice C. Whatley stated that a small farm would require a CMC of 1,000 people in order to be profitable. So, you might think that a city of 50,000 would be able to support 50 farms. However, this answer choice assumes that every single person in the city joins a CMC. The author never suggests that this is likely, so this is a pretty big assumption to make. Even if we accept this assumption, the numbers don't support "at least fifty." It should say "at most fifty," because this is the best possible scenario, with every single person in the city buying from these farms.